LSAT and Law School Admissions Forum

Get expert LSAT preparation and law school admissions advice from PowerScore Test Preparation.

 Administrator
PowerScore Staff
  • PowerScore Staff
  • Posts: 8916
  • Joined: Feb 02, 2011
|
#45219
Complete Question Explanation
(The complete setup for this game can be found here: lsat/viewtopic.php?t=15027)

The correct answer choice is (B)

This question creates a large VWQ Block, and asks us to determine which variable must be 6. The VWQ block, when combined with the Q – R sequence, produces the following sequencing chain:
PT76_O15 LG Explanations_Game #1_#6_diagram 1.png
Since P must be 4, the VWQ block must occupy the first three spaces in our setup, with the remaining variables (S, T, and R) occupying the last three spaces. Once again, the rotating ST Not Block is key: the only way to separate S and T is to place R in between them:
PT76_O15 LG Explanations_Game #1_#6_diagram 2.png
Since R must be sixth, answer choice (B) is the correct answer choice.
You do not have the required permissions to view the files attached to this post.

Get the most out of your LSAT Prep Plus subscription.

Analyze and track your performance with our Testing and Analytics Package.